Buscar

Resoluções do Curso de Análise - cap. 3

Faça como milhares de estudantes: teste grátis o Passei Direto

Esse e outros conteúdos desbloqueados

16 milhões de materiais de várias disciplinas

Impressão de materiais

Agora você pode testar o

Passei Direto grátis

Você também pode ser Premium ajudando estudantes

Faça como milhares de estudantes: teste grátis o Passei Direto

Esse e outros conteúdos desbloqueados

16 milhões de materiais de várias disciplinas

Impressão de materiais

Agora você pode testar o

Passei Direto grátis

Você também pode ser Premium ajudando estudantes

Faça como milhares de estudantes: teste grátis o Passei Direto

Esse e outros conteúdos desbloqueados

16 milhões de materiais de várias disciplinas

Impressão de materiais

Agora você pode testar o

Passei Direto grátis

Você também pode ser Premium ajudando estudantes
Você viu 3, do total de 27 páginas

Faça como milhares de estudantes: teste grátis o Passei Direto

Esse e outros conteúdos desbloqueados

16 milhões de materiais de várias disciplinas

Impressão de materiais

Agora você pode testar o

Passei Direto grátis

Você também pode ser Premium ajudando estudantes

Faça como milhares de estudantes: teste grátis o Passei Direto

Esse e outros conteúdos desbloqueados

16 milhões de materiais de várias disciplinas

Impressão de materiais

Agora você pode testar o

Passei Direto grátis

Você também pode ser Premium ajudando estudantes

Faça como milhares de estudantes: teste grátis o Passei Direto

Esse e outros conteúdos desbloqueados

16 milhões de materiais de várias disciplinas

Impressão de materiais

Agora você pode testar o

Passei Direto grátis

Você também pode ser Premium ajudando estudantes
Você viu 6, do total de 27 páginas

Faça como milhares de estudantes: teste grátis o Passei Direto

Esse e outros conteúdos desbloqueados

16 milhões de materiais de várias disciplinas

Impressão de materiais

Agora você pode testar o

Passei Direto grátis

Você também pode ser Premium ajudando estudantes

Faça como milhares de estudantes: teste grátis o Passei Direto

Esse e outros conteúdos desbloqueados

16 milhões de materiais de várias disciplinas

Impressão de materiais

Agora você pode testar o

Passei Direto grátis

Você também pode ser Premium ajudando estudantes

Faça como milhares de estudantes: teste grátis o Passei Direto

Esse e outros conteúdos desbloqueados

16 milhões de materiais de várias disciplinas

Impressão de materiais

Agora você pode testar o

Passei Direto grátis

Você também pode ser Premium ajudando estudantes
Você viu 9, do total de 27 páginas

Faça como milhares de estudantes: teste grátis o Passei Direto

Esse e outros conteúdos desbloqueados

16 milhões de materiais de várias disciplinas

Impressão de materiais

Agora você pode testar o

Passei Direto grátis

Você também pode ser Premium ajudando estudantes

Prévia do material em texto

Resoluções Curso de Análise Vol. 1 - Cap. 3
Elon Lages Lima
George Euzébio
Dezembro 2020
Capítulo 1
Números Reais
1. Dados a, b, c, d num corpo K, sendo b e d diferentes de zero, prove
que:
1º) a
b
+ c
d
= ad+bc
bd
2º) a
b
· c
d
= a·c
b·d
Resolução:
1º)
Como para todo n ∈ K existe n−1 ∈ K de modo que n · n−1 = 1K , então
sejam b−1, d−1 ∈ K tais que b · b−1 = d · d−1 = 1K , como ab = a · b
−1 e
c
d
= c · d−1, então temos que
a
b
+
c
d
= a · b−1 + c · d−1
= (a · b−1)(d · d−1) + (c · d−1)(b · b−1)
= (a · d)(b−1 · d−1) + (c · b)(d−1 · b−1)
= (a · d+ c · b)(b−1 · d−1)
= (a · d+ c · b)(b · d)−1
=
ad+ bc
bd
2º)
Seguindo como no item a), temos que
a
b
· c
d
= (a · b−1) · (c · d−1)
= (a · c) · (b−1 · d−1)
= (a · c) · (b · d)−1
=
a · c
b · d
2
3
2. Dado a 6= 0 num corpo K, põe-se, por definição, a0 = 1 e, se n ∈ N,
a−n = 1
an
ou seja, a−n = (an)−1. Prove:
1º) am · an = am+n
2º) (am)n = amn
Resolução:
1º) Como por definição a0 = 1, então ∀m ∈ N temos que am = am+0, por
outro lado, am = am · 1 = am · a0, logo concluímos que am+0 = am · a0.
Sendo m, n ∈ N, como am possui m fatores a, por definição, então podemos
dizer que am+1 possui m+ 1 fatores a e podemos reescrever am+1 = am · a =
am · a1. Supondo que para algum n natural seja válido am+n = am · an,
então ao multiplicarmos ambos os membros da igualdade por a, obtemos
am+n · a = (am · an) · a = am · an · a, onde aplicando novamente a definição
anterior, temos am+(n+1) = a(m+n)+1 = am · (an · a1) = am · an+1. O resultado
é igualmente provado para os casos em que m e n são negativos, uma vez que
−m e −n pertencerão à N e, por definição, a−n = (an)−1.
2°) Para provarmos que (am)n = amn, usaremos indução em n, considerando
nm > 0. De fato, como (am)1 = am = am·1, então vamos supor que, para
um dado n natural, (am)n = amn. Desse modo, temos que (am)n+1 = (am)n ·
(am)1 = amn · am = amn+m = am(n+1) e podemos ver que é válido para todo
natural. No caso em que nm < 0, basta escrever a−nm = (anm)−1 e usar o
resultado anterior.
3. Se x1
y1
= x2
y2
= . . . = xn
yn
num corpo K, prove que, dados a1, . . . , an ∈ K
tais que a1y1 + . . .+ anyn 6= 0, tem-se a1x1+...+anxna1y1+...+anyn =
x1
y1
.
Resolução:
De fato, sendo xi
yi
= k, com 1 ≤ i ≤ n, podemos reescrever xi = yik, e
como aixi = aiyik, temos que
∑n
i aixi = (
∑n
i aiyi)k. Como já vimos que∑n
i aiyi 6= 0, então
∑n
i aixi∑n
i aiyi
= k =⇒ a1x1+...+anxn
a1y1+...+anyn
= x1
y1
.
4. Sejam K, L corpos. Uma função f : K → L chama-se um homomor-
fismo quando se tem f(x + y) = f(x) + f(y) e f(x · y) = f(x) · f(y),
quaisquer que sejam x, y ∈ K.
i) Dado um homomorfismo f : K → L, prove que f(0) = 0.
ii) Prove também que, ou f(x) = 0 para todo x ∈ K, ou então
f(1) = 1 e f é injetivo.
4 CAPÍTULO 1. NÚMEROS REAIS
Resolução:
i) Podemos observar que, sendo f um homomorfismo, é válido que f(0) =
f(0 + 0) = f(0) + f(0) =⇒ f(0) = 0.
ii) Temos obviamente que para todo x ∈ K é válido f(x) = f(x · 1) =
f(x)·f(1). Ou seja, temos então que para todo x ∈ K, f(x) = f(x)·f(1) =⇒
f(x)(1 − f(1)) = 0 o que nos dá que f(x) = 0 para todo x ou f(1) = 1. A
partir da segunda igualdade podemos concluir que f é injetiva, uma vez que
f(1) = 1 e para todo x ∈ K, f(x+ 1) = f(x) + 1.
5. Sefa f : Q→ Q um homomorfismo. Prove que, ou f(x) = 0 para todo
x ∈ Q ou então f(x) = x para todo x ∈ Q.
Resolução:
Pelo item ii) da questão anterior, como Q é um corpo, então garantimos que
para todo x ∈ Q ou f(x) = 0 ou f(x) é injetiva. Sendo f injetiva, então como
f(1) = 1 e f(m + 1) = f(m) + 1, podemos usar o 2◦ princípio de indução
para obter que f(m) = m para todo m ∈ N. A extensão para Z se faz pelo
seguinte fato: como f(m) = f(m + 0) = f(m) + f(0) =⇒ f(0) = 0, então
0 = f(0) = f(m + (−m)) = f(m) + f(−m) =⇒ f(−m) = −f(m) = −m,
para todo m ∈ N. Para complementarmos o conjunto f(Q), mostraremos
que é válido f
(
m
n
)
= m
n
. Temos então que para todo m ∈ Z e n ∈ Z∗ é válido
f(m) = f
(
m · n
n
)
= f
(m
n
· n
)
= f
(m
n
)
· f(n)
onde concluímos que f
(
m
n
)
= f(m)
f(n)
= m
n
para todom ∈ Z e n ∈ Z∗. Portanto,
para todo x ∈ Q, tem-se f(x) = x.
7. Seja p um natural primo. Para cada inteiro m, indiquemos como m
o resto da divisão de m por p. No conjunto Zp = {0, 1, . . . , p − 1}
definamos duas operações: uma adição⊕ e uma multiplicação�, pondo
m ⊕ n = m+ n e m � n = m · n. Prove que a função f : Z → Zp,
definida por f(n) = n, cumpre f(m + n) = f(m) ⊕ f(n) e f(m ·
n) = f(m) � f(n). Conclua que ⊕ e � são comutativas, associativas,
vale a distributividade, existem 0 e 1. Observe que dados m, n ∈ Zp,
m · n = 0 =⇒ m = 0 ou n = 0. Conclua que Zp é um corpo.
Resolução:
Usando congruência modular, temos que m ≡ m(mod p) e n ≡ n(mod p)
implicam nos resultados m + n ≡ m + n(mod p) e m · n ≡ m · n(mod p),
5
e pela definição de congruência modular, temos que m + n deixa o mesmo
resto que m+ n na divisão por p, ou seja, m+ n = m+ n, assim como m · n
também deixa o mesmo resto que m · n na divisão por p, garantindo que
m · n = m · n.
Com o resultado acima, garantimos que f(m + n) = m+ n = m+ n =
f(m) + f(n) = f(m) ⊕ f(n) e f(m · n) = m · n = m · n = f(m) · f(n) =
f(m) � f(n). Como a congruência modular é assosciativa, comutativa e é
válido a distributividade podemos concluir que também é válido para f .
Sendo pZ o conjunto dos mútiplos de p, se x ∈ pZ, então m = 0, logo para
todo x ∈ pZ temos que f(x+ n) = f(n), ou seja, existe um elemento neutro
em Zp.
Definamos o conjunto pZ+1 = {x ∈ Z;x = αp+1}. Temos que pZ+1 6= ∅,
uma vez que 1 ∈ pZ + 1. Desse modo, para todo x ∈ pZ + 1, x = 1, logo
temos que f(m ·x) = f(m) ·f(x) = f(m) ·1 = f(m), para todo m ∈ Z. Como
os conjuntos x são classes de equivalências, não se faz necessário mostrar a
unicidade dos elementos neutros da adição e multiplicação, uma vez que todo
elemento da classe x é um representante da mesma.
Para concluirmos, sem�n = 0 então devemos ter quem · n = 0 =⇒ m·n ≡
0(mod p), ou seja, p | m ·n e como p é primo, então p | m ou p | n, mostrando
então que m = 0 ou n = 0.
Como pudemos ver Zp munido das operações ⊕ e � é um corpo, uma vez que
a função f como foi definida possui as propriedades associativa, comutativa,
distibutiva e existe em Zp elementos neutros para a adição e multiplicação,
além do mais, o produto de dois termos de Zp só é nulo se um dos fatores é
nulo.
8. Seja K um conjunto onde são válidos todos os axiomas de corpo, salvo
a existência de inverso multiplicativo.
i) Dado a 6= 0 em K, prove que a função f : K → K, definida por
f(x) = ax, é uma bijeção se, e comente se, a possui inverso.
ii) Mostre que f é injetiva se, e somente se, vale a lei do corte para
a.
iii) Conclua que, se K é finito, a lei do corte é equivalente à existência
de inverso para cada elemento não-nulo de K.
Resolução:
(=⇒)
Vamos mostrar que f é injetiva. De fato, dados quaisquer x, y ∈ K, temos
6 CAPÍTULO 1. NÚMEROS REAIS
que
f(x) = f(y) =⇒ ax = ay
=⇒ a(x− y) = 0, a 6= 0
=⇒ x = y
e f é injetiva. Para a sobrejetividade, observamos que para todo x ∈ K,
temos que x = x · 1 = x · (a−1 · a) = (x · a−1) · a = a · x′, ou seja, para todo
x ∈ K existe um x′ ∈ K tal que f(x′) = x, portanto f é sobrejetiva.
(⇐=)
Sendo f uma bijeção, então existe uma função inversa f−1 tal que para
todo x, y ∈ K, f−1(y) = x ⇐⇒ f(x) = y, ou seja, f−1(y) = f−1(f(x)) =
f−1(ax) = x. Sendo f−1(y) = by, então
f−1(ax) = b(ax) = x =⇒ (ba)x = x
=⇒ (ba− 1)x = 0, tomando x 6= 0
=⇒ ba = 1
portanto concluímos que b = a−1 e que a possui inverso.
ii) De fato, como fizemos na primeira parte do item anterior, a definição de
lei do corte é a própria injetividade de f .
iii) Se K é finito e vale a lei do corte, então f é bijetora uma vez que todo
elemento de K terá uma imagem por f , portanto sobrejetiva, e pelo item
ii) f é injetiva. Assim concluímos que todo elemento não nulo de K existe
inverso.
9. Explique por que as operações usuais não tornam corpos o conjunto Z
dos inteiros nem o conjuntoQ[t] dos polinômios de coeficientes racionais.
Resolução:
Nem todo elemento de Z ou Q[t] possui inverso multiplicativo.10. Num corpo ordenado K, prove que a2 + b2 = 0⇐⇒ a = b = 0.
Resolução:
Supondo a 6= 0, então a2 > 0 e somando b2 em ambos os lados da desigualdade
temos que a2+b2 > b2. Como b2 ≥ 0, então a2+b2 > b2 ≥ 0 =⇒ a2+b2 > 0.
O mesmo ocorre se considerarmos inicialmente b 6= 0.
11. Seja P o conjunto dos elementos positivos de um corpo ordenado K.
i) Dado um número natural n, prove que a função f : P → P ,
definida por f(x) = xn, é monótona crescente (isto é, x < y =⇒
f(x) < f(y)).
7
ii) Dê um exemplo em que a função não é sobrejetiva.
iii) Prove que f(P ) não é um conjunto limitado superiormente de K.
Resolução:
i)
Supondo x < y, vamos mostrar que xn < yn para todo natural usando
indução sobre n. Tomando como hipótese que xn < yn para um dado n ∈ N,
então as desigualdades xn+1 < ynx e ynx < yn+1 implicam em xn+1 < yn+1.
Com isso, para todo n natural, temos que x < y =⇒ f(x) < f(y), portanto
f é monótona crescente.
ii) Tomando n = 3, podemos usar a função f(x) = (x− 1)3
iii) Tomando P ⊂ K como sendo o conjunto ordenado P = {k1 < k2 <
. . . < kn < . . .} indexado convenientemente. Seja então α ∈ P tal que
f(α) = supf(P ), desse modo, f(P ) = {f(k1), f(k2), . . . , f(kn), . . . , f(α)}.
Seja então β ∈ K tal que α < β, como f é monótona crescente, temos que
f(α) < f(β), como f(β) > 0 =⇒ f(β) ∈ f(P ), concluindo assim que f(P )
não é limitado.
12. SejamX um conjunto qualquer eK um corpo. Indiquemos com F(X;K)
o conjunto de todas as funções f : X → K. Definamos em F(X;K)
as operações de adição e de multiplicação de modo natural: dadas
f, g : X → K, as funções f + g : X → K e f · g : X → K são dadas
por (f + g)(x) = f(x) + g(x) e (f · g)(x) = f(x) · g(x). Verifique quais
dos axiomas de corpo são válidos e quais não são válidos no conjunto
F(X;K), relativamente a estas operações.
13. Sejam x, y elementos positivos de um corpo ordenado K. Tem-se x <
y ⇐⇒ x−1 > y−1. Prove também que x > 0⇐⇒ x−1 > 0.
Resolução:
De fato, x < y =⇒ xx−1 < yx−1 =⇒ y−1(xx−1) < y−1(yx−1) =⇒ y−1 <
x−1. Como xx−1 > 0 e x > 0, então x−1 > 0.
14. Seja a um elemento positico de um corpo ordenado K. Definamos
f : Z → K pondo f(n) = an. (Veja o Exercício 2.) Prove que f é
crescente se a > 1, decrescente se a < 1 e constante se a = 1.
Resolução:
Se a > 1 ou a < 1, basta multiplicarmos as desigualdades por an, onde
obteremos an+1 > an ou an+1 < an,para todo n natural. Se a = 1, então
an = 1 para todo n natural.
8 CAPÍTULO 1. NÚMEROS REAIS
15. Dados x 6= 0 num corpo ordenado K e n ∈ N qualquer, prove que
(1 + x)2n > 1 + 2n · x.
Resolução:
Basta tomarmos (1+x)2n = [(1+x)2]n = [1+2x+x2]n > (1+2x)n. Aplicando
a desigualdade de Bernoulli, temos que (1 + 2x)n > 1 + n · (2x) = 1 + 2n · x,
e concluímos que (1 + x)2n > 1 + 2n · n.
16. Se n ∈ N e x < 1 num corpo ordenado K, prove que (1−x)n ≥ 1−nx.
Resolução:
Basta usarmos indução em n. É fácil ver que a desigualdade é verdadeira
para n = 1. Supondo ser válida para um dado n natural, então mostremos ser
válida para n+1. De fato, temos que (1−x)n+1 = (1−x)(1−x)n =⇒ (1−
x)n+1 > (1−x)(1−nx). Como (1−x)(1−nx) = 1−(n+1)x+x2 > 1−(n+1)x,
então concluímos que (1− x)n+1 > 1− (n+ 1)x e que a desiguldade é válida
para todo n ∈ N.
17. Num corpo ordenado, se a e a+ x são positivos, prove que (a+ x)n ≥
an + n · an−1 · x. enuncie e demonstre desigualdades análogas às dos
Exercícios 15 e 16, com a em vez de 1.
Resolução:
Basta aplicar indução em n usando a expansão binomial (a+x)n =
∑n
p=0
(
n
p
)
an−pxp,
onde teremos (a+x)n = an+n ·an−1 ·x+
∑n
p=2
(
n
p
)
an−pxp ≥ an+n ·an−1 ·x.
18. Sejam a, b, c, d elementos de um corpo ordenado K, onde b e d são
positivos. Prove que a+c
b+d
está compreendido entre o menor e o maior
elemento dos elementos a
b
e c
d
. Generalize: mostre que a1+...+an
b1+...+an
está
compreendido entre o menor e o maior dos elementos a1
b1
, . . . , an
bn
, desde
que b1, . . . , bn sejam todos positivos.
Resolução:
Temos que min(a, c) < a+c
2
< max(a, c) e min(b, d) < b+d
2
< max(b, d),
então, dividindo a primeira desigualdade por b+d
2
, temos que 2min(a,c)
b+d
<
a+c
b+d
< 2max(a,c)
b+d
=⇒ min(a,c)
max(b,d)
< a+c
b+d
< max(a,c)
min(b,d)
, logo podemos concluir que
min(a
′
b′
, c
′
d′
) < a+c
b+d
< max(a
′
b′
, c
′
d′
), onde a′
b′
, c
′
d′
são todas as frações onde o nu-
merador é formado por a ou c e o denominador é formado por b ou d. A
generalização é garantida a partir da generalização da média aritmética de
um conjunto de valores.
9
19. Dados x, y num corpo ordenado K, com y 6= 0, prove que |x · y−1| =
|x| · |y|−1, ou seja, |x
y
| = |x||y| .
Resolução:
Como, por definição, |x| =
√
x2 podemos então escrever |x·y−1| =
√
(x · y−1)2 =√
x2 · (y−1)2 =
√
x2 ·
√
(y−1)2 =
√
x2 · (
√
y2)−1, uma vez que
√
(y−1)2 =
((y−1)2)
1
2 e podemos permutar os expoentes. Assim concluímos que |x·y−1| =
|x| · |y|−1.
20. Prove por indução que, dados x1, . . . , xn num corpo ordenado K, tem-
se |x1 + . . .+ xn| ≤ |x1|+ . . .+ |xn| e |x1 · x2 . . . xn| = |x1| · |x2| . . . |xn|.
Resolução:
Para n = 2 basta verificar o teorema 2 do capítulo. Supondo ser válido para
um dado n natural, então mostremos para n+ 1, ou seja
|x1 + x2 + . . .+ xn + xn+1| = |(x1 + x2 + . . .+ xn + xn+1)|
≤ |x1 + x2 + . . .+ xn|+ |xn+1|
como por hipótese |x1 + . . . + xn| ≤ |x1| + . . . + |xn|, então concluímos que
|x1 + . . .+ xn+ xn+1| ≤ |x1|+ . . .+ |xn|+ |xn+1. Logo a propriedade é válida
para todo n natural. A segunda propriedade é demonstrada de modo análogo,
basta observar que |x1 ·x2| =
√
(x1 · x2)2 =
√
x21 · x22 =
√
x21 ·
√
x22 = |x1|·|x2|
e supondo ser válido para um dado n, mostramos para n + 1 escrevendo
|x1 · x2 . . . xn · xn+1| = |(x1 · x2 . . . xn) · xn+1| = |x1 · x2 . . . xn| · |xn+1| =
|x1| · |x2| . . . |xn| · |xn+1|, logo é válido para todo n natural.
22. Prove que, para todo x num corpo ordenado K, tem-se
i) |x− 1|+ |x− 2| ≥ 1.
ii) |x− 1|+ |x− 2|+ |x− 3| ≥ 2
Resolução:
i)
De maneira simples temos que
|2− 1| = 1
|2− x+ x− 1| = 1
|2− x|+ |x− 1| ≥ 1
|x− 1|+ |x− 2| ≥ 1
10 CAPÍTULO 1. NÚMEROS REAIS
ii)
De modo análogo, temos que
|3− 1| = 2
|3− x+ x− 1| = 2
|3− x|+ |x− 1| ≥ 2
|x− 3|+ |x− 1| ≥ 2
como |x−2| ≥ 0 para todo x ∈ K, então podemos concluir que |x−1|+ |x−
2|+ |x− 3| ≥ 2.
23. Dados a, b, � num corpo ordenado K, prove que
|a− b| < � =⇒ |b| − � < |a| < |b|+ �.
Conclua que |a− b| < � =⇒ a < |b|+ �
Resolução:
Basta observar que ||a|− |b|| ≤ |a− b| < � =⇒ ||a|− |b|| < � e pela definição
de módulo, −� < |a| − |b| < � =⇒ |a| < |b| + �. Para concluirmos, como
a ≤ |a|, então |a− b| < � =⇒ a < |b|+ �
24. Prove que, num corpo ordenado K, as seguintes afirmações são equiv-
alentes:
(i) K é arquimediano;
(ii) Z é ilimitado superior e inferiormente;
(iii) Q é ilimitado superior e inferiormente.
Resolução:
Um corpo K é arquimediano quando dados a, b,∈ K, com a > 0, existe
n ∈ N tal que b < na.
(i) =⇒ (ii)
Sendo K arquimediano e Z ⊂ K limitado superiormente, então existe a =
supZ tal que para todo x ∈ Z, x ≤ a. Seja então x0 um inteiro positivo,
como K é arquimediano, então existe n ∈ N tal que a < nx0 e como nx0 ∈ Z,
concluímos que a existância desse supremo é um absurdo! Logo Z é ilimitado
superiormente. Para prova de que Z é ilimitado inferiormente é análoga.
(ii) =⇒ (iii)
Sendo Z ilimitado superiormente e inferiormente, então dados a, b ∈ Z, com
a < b, podemos tomar um inteiro c de modo a obter a
c
< b
c
ou b
c
< a
c
, tornando
11
assim Q ilimitado superior e inferiormente.
(iii) =⇒ (i)
Como Q é ilimitado, então para todo n ∈ N existem a, b ∈ Q, com a < b,
tais que 1
n
< a
b
. Logo podemos concluir que b < na e K é arquimediano.
25. Prove que um corpo ordenado K é arquimediano se, e somente se, para
todo � > 0 em K, existe n ∈ N tal que 1
2n
< �.
Resolução:
Como para todo n ∈ N é válido que n < 2n, então 1
2n
< 1
n
e K será arquime-
diano se, e somente se, � > 1
n
. Desse modo garantimos a existência de n e �.
26. Seja a > 1 num corpo arquimediano K. Considere a função f : Z→ K,
definida por f(n) = an. Prove as seguintes afirmações:(i) f(Z) não é limitado superiormente;
(ii) inf f(Z) = 0.
Resolução:
(i)
Como foi mostrado no exercício 14, se a > 1, então f é monótona crescente.
Se f(Z) fosse limitado superiormente, então deve existir um n0 ∈ K, n0 > n
para todo n ∈ Z, talque f(n) ≤ f(n0), ou seja, an ≤ an0 . Como K é
arquimediano existe m ∈ N de modo que n0 < mn, e como f é monótona
crescente, concluímos que f(n0) < f(mn) =⇒ an0 < amn, mostrando assim
que f(Z) é ilimitado superiormente.
(ii)
Como Z é ilimitado superior e inferiormente, então tomando −n > 0, temos
que a−n = 1
an
. Como a > 1, então 0 < 1
an
< 1,ou seja, 0 é uma cota
inferior de f . Seja 0 < f(n1) = inff(Z), então f(n1) ≤ f(nk), para todo nk
inteiro. Como K é arquimediano, existe um inteiro m tal que −n0 < −mnk,
e usando a monotonicidade de f , temos que 0 < f(mnk) < f(n1). Portanto,
concluímos que inff(Z) = 0.
27. Sejam a racional diferente de zero, e x irracional. Prove que ax e a+x
são irracionais. Dê exemplo de dois números irracionais x, y tais que
x+ y e x · y são racionais.
Resolução:
Vamos supor que a+ x e ax são racionais, desse modo a+ x = p1
q1
=⇒ x =
p1
q1
− a e ax = p2
q2
=⇒ x = p2
aq2
, ou seja, em ambas as situações concluiríamos
12 CAPÍTULO 1. NÚMEROS REAIS
que x é racional, o que é um absurdo contra a hipótese de que x é irracional.
Portanto, temos que x + a e ax são irracionais. Para os exemplos, basta
tomar y = −x = √p, com p primo.
28. Sejam a, b, c e d números racionais. Prove que a+ b
√
2 = c+d
√
2⇐⇒
a = c e b = d.
Resolução:
É fácil ver que se a = c e b = d, a igualdade é verdadeira. Do contrário, se
a 6= c e b 6= d, então
a+ b
√
2 = c+ d
√
2 =⇒ a− c = (d− b)
√
2
=⇒
√
2 =
a− c
d− b
(1.1)
o que é um absurdo, pois
√
2 não é racional. Portanto, devemos ter a = c e
b = d.
29. Prove que o conjunto K dos números reais da forma a + b
√
2, com
a e b racionais, é um corpo relativamente às operações de adição e
multiplicação de números reais. Examine se o mesmo ocorre com os
números da forma a+ b 3
√
2, com a, b ∈ Q.
Resolução:
Sejam quaisquer α, β ∈ K de modo que α = a + b
√
2 e β = c + d
√
2, com
a, b, c, d ∈ Q, é fácil observar que α + β, α · β ∈ K uma vez que α + β =
(a+c)+(b+d)
√
2 e α·β = (ac+2bd)+(ad+bd)
√
2. Temos que 0, 1 ∈ K, basta
tomar a = b = 0 e a = 1, b = 0, respectivamente. Todo elemento não nulo
possui um inverso da forma a−b
√
2
a2−2b2 . As propriedades comutativa, associativa e
distributiva são garantidas uma vez que a, b, c, d ∈ Q. O mesmo ocorre com
os números da forma a+ b 3
√
2, porém, o inverso terá a forma (a
2+b2 3
√
2
2
)−ab 3
√
2
a3+2b3
.
30. Sejam a, b números racionais positivos. Prove que
√
a+
√
b é racional se,
e somente se,
√
a e
√
b forem ambos irracionais. (Sugestão: multiplique
por
√
a−
√
b)
Resolução:
(=⇒)
Trivial
(⇐=)
13
Sendo
√
a+
√
b racional, então
√
a+
√
b = p
q
com p, q ∈ Q. Multiplicando a
última igualdade por
√
a−
√
b obtemos
a− b = p
q
·
√
a−
√
b =⇒
√
a =
q
p
· (a− b+
√
b)
Logo, se
√
b é racional, então
√
a também é, do contrário, se
√
b não é
racional, então
√
a+
√
b não será racional, contrariando nossa hipótese.
31. Sejam X ⊂ R não-vazio, limitado superiormente, e c um número real.
Tem-se c ≤ supX se, e somente se, para cada � > 0 real dado pode-se
achar x ∈ X tal que c − � < x. Enuncie e demonstre um resultado
análogo com inf em vez de sup.
Resolução:
Sendo c ∈ R, podemos sempre tomar um � > 0 tal que |x − c| < �, para
cada x ∈ X. Temos que se c < x, então c ≤ supX, caso contrário, se x < c,
então como R é arquimediano, existe um natural n tal que c < nx. Desse
modo, |x′ − c| < �′ =⇒ c − �′ < x′, com x′ = nx. Assim concluímos que
c ≤ supX ⇐⇒ x′ ∈ X.
32. Seja X =
{
1
n
;n ∈ N
}
. Prove que inf X = 0.
Resolução:
Vamos supor, por contradição, que existen0 ∈ N tal que 0 < n0 = inf X.
Como N é ilimitado superiormente, então existe n natural tal que n0 < n =⇒
1
n
< 1
n0
, nos mostrando que n0 não pode ser uma cota inferior de X, portanto,
inf X = 0.
33. Sejam A ⊂ B conjuntos não-vazios limitados de números reais. Prove
que inf B ≤ inf A ≤ supA ≤ supB.
Resolução:
Como R é ordenado, basta tomarmos B = {x1 ≤ x2 ≤ . . . ≤ xn}, com
xi ∈ R, 1 ≤ i ≤ n. Desse modo o resultado segue intuitivamente.
34. Sejam A, B conjuntos não-vazios de números reais, tais que x ∈ A, y ∈
B =⇒ x ≤ y. Prove que supA ≤ inf B. Prove que supA = inf B
se, e somente se, para todo � > 0 dado, podem-se obter x ∈ A e y ∈ B
tais que y − x < �.
14 CAPÍTULO 1. NÚMEROS REAIS
Resolução:
Como x ≤ supA e inf B ≤ y, para todo x ∈ A e y ∈ B, então supA ≤
inf B ou inf B ≤ supA. Se inf B ≤ supA, então inf B não seria a maior
cota inferior de B e supA não poderia ser a menor cota superior de A.
Logo devemos ter supA ≤ inf B. Pelo exercício 31, se y − x < �, então
inf B ≤ supA e como foi mostrado anteriormente, temos que supA ≤ inf B.
Portanto, supA = inf B.
35. Dado A ⊂ R não-vazio, limitado inferiormente, seja −A = {−x;x ∈
A}. Prove que −A é limitado superiormente e que sup(−A) = −inf A.
Resolução:
Sendo A não-vazio e limitado inferiormente, então para todo x ∈ A temos
que inf A ≤ x =⇒ −x ≤ −inf A, nos mostrando que −A é limi-
tado superiormente. Podemos observar que −x ≤ sup(−A) ≤ inf A =⇒
inf A ≤ sup(−A) ≤ x, e essa implicação nos mostra que devemos ter
sup(−A) = inf A, do contrário, se sup(−A) > inf A teríamos que inA
não seria a maior cota inferior do conjunto A.
36. Seja A ⊂ R não vazio, limitado. Dado c > 0, seja c ·A = {c ·x;x ∈ A}.
Prove que c·A é limitado e que sup(c·A) = c·supA, inf(c·A) = c·inf A.
Enuncia e demosntre o que ocorre quando c < 0.
Resolução:
Sendo A limitado, então inf A ≤ x ≤ supA =⇒ c · inf A ≤ cx ≤ c · supA,
portanto, c · A é limitado. Supondo que a = inf A, então para todo x ∈ A,
a ≤ x =⇒ ca ≤ cx. Se existesse a0 tal que ca ≤ ca0 ≤ cx, implicaria em
a ≤ a0 ≤ x, e como a é a maior cota inferior de A, assim concluímos que
a = a0 e que ca = inf(c ·A), ou seja, inf(c ·A) = c · inf A. De modo análogo
é mostrado que sup(c · A) = c · supA.
37. Dados A, B ⊂ R não-vazios e limitados, seja A + B = {x + y;x ∈
A, y ∈ B}. Prove:
i) A+B é limitado;
ii) sup(A+B) = supA+ supB;
iii) inf(A+B) = inf A+ inB;
iv) Enuncie e demonstre resultados análogos supondo apenas A e B
limitados superiormente (ou A e B limitados inferiormente).
15
Resolução:
i) Basta observar que para todos x ∈ A e y ∈ B, inf A ≤ x ≤ supA e
inf B ≤ y ≤ supB implicam em inf A+ inf B ≤ x+ y ≤ supA+ supB.
ii) Como A, B ⊂ R são limitados, então x ≤ supA e y ≤ supB implicam
que x + y ≤ supA + supB, logo supA + supB é uma cota superior de
A + B e podemos considerar sup(A + B) ≤ supA + supB. Temos que
x+y ≤ sup(A+B) =⇒ x ≤ sup(A+B)−y, então sup(A+B)−y é uma cota
superior deA, portanto, supA ≤ sup(A+B)−y =⇒ y ≤ sup(A+B)−supA,
o que também nos leva a afirmar que sup(A+B)−supA é uma cota superior
de B e, portanto, temos supB ≤ sup(A+B)− supA =⇒ supA+ supB ≤
sup(A+B), concluindo assim que supA+ supB = sup(A+B).
iii)A demonstração dessa propriedade é análoga a demonstração feita em ii),
basta considerar o ínfimo de cada conjunto.
38. Seja X ⊂ R. Uma função f : X → R chama-se limitada quando sua
imagem f(X) ⊂ R é um conjunto limitado. Neste caso define-se o
sup f como o supremo do conjunto f(X). (Às vezes se escreve sup f(x)
ou sup f)
i) Prove que a soma de duas funções limitadas f, g : X → R é uma
função limitada f + g : X → R.
ii) Mostre que (f + g)(X) ⊂ f(X) + g(X), na notação do Exercício
37.
iii) Conclua que sup(f + g) ≤ sup f + sup g e que inf(f + g) ≥
inf f + inf g.
iv) Considerando as funções f, g : [−1,+1]→ R, definidas por f(x) =
x e g = −x, mostre que se pode ter sup(f + g) < sup f + sup g e
inf(f + g) > inf f + inf g.
Resolução:
i) Sendo f, g : X → R limitadas, então f(X) e g(X) são limitados, portanto,
basta aplicar o item i) do exercício anterior.
ii) Sendo f(X) + g(X) = {f(x) + g(y); f(x) ∈ f(X) e g(y) ∈ g(X)}, se
α ∈ (f + g)(X), então existe x ∈ X tal que α = (f + g)(x) = f(x) +
g(x) =⇒ α ∈ f(X) + g(X).A inclusão inversa não é verdadeira, pois dado
f(x)+g(y) = β ∈ f(X)+g(X),com x 6= y, implica β 6∈ (f+g)(X), portanto,
concluímos que (f + g)(X) ⊂ f(X) + g(X).
iii) Como f(x) ≤ sup f(X) e g(x) ≤ sup g(X) =⇒ (f + g)(x) = f(x) +
g(x) ≤ sup f(X)+sup g(X), então sup f(X)+sup g(X) é uma cota superior
de (f + g)(X), portanto, sup(f + g) ≤ sup f + sup g. A desigualdade inversa
não é verdadeira devido ao item anterior que nos mostra que (f + g)(X) ⊂
16 CAPÍTULO 1. NÚMEROS REAIS
f(X)+g(X), sendo (f+g)(X) próprio. Para o ínfimo o raciocínio é análogo.
iv) Como temos que sup f = sup g = 1, inf f = inf g = −1 e sup(f + g) =
inf(f+g) = 0, então sup(f+g) < sup f+sup g e inf(f+g) > inf f+inf g.
39. Sejam A, B conjuntos de números reais positivos. Definamos A · B =
{x·y;x ∈ A e y ∈ B}. Prove que seA eB forem limitados entãoA·B
é limitado, sendo sup(A·B) = supA·supB e inf(A·B) = inf A·inf B.
Resolução: Sendo A e B limitados, para todos x ∈ A e y ∈ B temos que
inf A ≤ x ≤ supA e inf B ≤ y ≤ supB de modo que ao multiplicarmos a
primeira desigualdade por y, obtemos inf A · y ≤ x · y ≤ supA · y. Portanto,
A · B é limitado. Como inf B ≤ y ≤ supB, então inf A · y ≤ x · y ≤
supA · y =⇒ inf A · inf B ≤ x · y ≤ supA · supB. Vemos então que
inf A · inf B é uma cota inferior de A · B, enquanto supA · supB é uma
cota superior. Se existem reais α = inf(A · B), β = sup(A · B), então
inf A · inf B ≤ α ≤ x · y ≤ β ≤ supA · supB, e ao dividirmos toda essa
desigualdade por y, obtemos inf A · inf B
y
≤ α
y
≤ x ≤ β
y
≤ supA · supB
y
. Como
inf B
y
≥ 1 e supB
y
≥ 1, garantimos que inf A ≤ α
y
≤ x ≤ β
y
≤ supA. A
desigualdade anterior gera o absurdo de que inf A não pode ser o ínfimo do
conjunto A assim como supB não pode ser o supremo do conjunto B. Logo
concluímos que inf(A ·B) = inf A · inf B e sup(A ·B) = supA · supB.
40. i) Prove que o produto de duas funções limitadas f, g : X → R é uma
função limitada f · g : X → R.
ii) Mostre que (f · g)(X) ⊂ f(X) · g(X).
iii) Conclua que, se f e g forem ambas positivas, tem-se sup(f · g) ≤
sup f · sup g e inf(f · g) ≤ inf f · inf g
iv) Dê exemplo em que valham as desigualdades estritas.
v) Mostre porém que para toda f positiva tem-se sup(f 2) = [sup f ]2.
Resolução:
i) Basta aplicar o exercício anterior tomando A = f(X) e B = g(X).
ii) Usando a notação do exercício 39, (f · g)(X) = {f(x) · g(x); f(x) ∈
f(X) e g(x) ∈ g(X)}, e se α ∈ (f · g)(X), então existe x ∈ X tal que
α = (f ·g)(x) = f(x) ·g(x), ou seja, α ∈ f(X) ·g(X) e, portanto, (f ·g)(X) ⊂
f(X) · g(X). Como f(X) · g(X) = {f(x) · g(y); f(x) ∈ f(X) e g(y) ∈ g(X)},
todo produto f(x) · g(y) 6∈ (f · g)(X) sempre que x 6= y. Portanto, a inclusão
inversa não ocorre.
iii) Como f(x) ≤ sup f(X) e g(x) ≤ sup g(X), sendo f, g ambas positi-
vas, então (f · g)(x) = f(x) · g(x) ≤ sup f(X) · sup g(X), ou seja, sup f(X) ·
sup g(X) é uma cota superior de (f ·g)(X), portanto, sup(f ·g) ≤ sup f ·sup g.
17
A desigualdade inversa não é verdadeira devido ao item anterior que nos
mostra que (f · g)(X) ⊂ f(X) · g(X), sendo (f + g)(X) próprio. Para o
ínfimo o raciocínio é análogo.
iv) Basta tomar f, g : [1, 2]→ R, com f(x) = x e g(x) = 1
x
v) Basta tomar f = g pelo resultado do item iii).
42. Seja f(x) = a0+a1x+. . .+anxn um polinômio com coeficientes inteiros.
i) Se um número racional p
q
(com p e q primos entre si) é tal que
f(p
q
) = 0, prove que p divide a0 e q divide an.
ii) Conclua que, quando an = 1, as raízes reais de f são inteiras ou
irracionais. Em particular, examinando xn − a = 0, conclua que,
se um número inteiro a > 0 não possui n-ésima raiz inteira, então
n
√
a é irracional.
iii) Use o resultado geral para provar que
√
2 + 3
√
2 é irracional.
Resolução:
i) sendo f
(
p
q
)
= 0, temos então que
an
(
p
q
)n
+ an−1
(
p
q
)n−1
+ . . .+ a1
(
p
q
)
+ a0 = 0
an
(
p
q
)n
+ an−1
(
p
q
)n−1
+ . . .+ a1
(
p
q
)
= −a0
p ·
(
anp
n + anp
n−1q + . . .+ a1q
n−1) = (−a0)qn
p · α = a0(−qn)
com α = (anpn + anpn−1q + . . .+ a1qn−1), isso nos mostra que p divide o
produto a0(−qn), pois existe um inteiro α ∈ Z tal que p · α = a0(−qn), e
como p e q são primos entre si, concluímos que p divide a0. Analogamente
temos
an
(
p
q
)n
+ an−1
(
p
q
)n−1
+ . . .+ a1
(
p
q
)
+ a0 = 0
an−1
(
p
q
)n−1
+ . . .+ a1
(
p
q
)
= −an
(
p
q
)n
q
(
an−1p
n−1 + . . .+ a1pq
n−2 + a0q
n−1) = an(−pn)
q · β = an(−pn)
logo existe um inteiro β = (an−1pn−1 + . . .+ a1pqn−2 + a0qn−1) tal que q ·β =
an(−pn), portanto q divide o produto an(−pn), e como p e q são primos entre
18 CAPÍTULO 1. NÚMEROS REAIS
si, garantimos que q divide an.
ii) Como visto no item anterior, q | an e se an = 1, então q | 1 =⇒ q = ±1.
Desse modo, se p
(
p
q
)
= 0, então f(p) = 0 ou f(−p) = 0, com p ∈ Z.
Do contrário f admite uma raiz irracional α.Sendo f(x) = xn − a, se f
não admite solução inteira, seja α um irracional tal que αn − a = 0. Essa
igualdade nos mostra que (α− n
√
a)(αn−1 + αn−2a+ . . .+ an−1) = 0, e como
αn−1 + αn−2a+ . . .+ an−1 6= 0, uma vez que a > 0, temos que α = n
√
a, logo
n
√
a é irracional.
iii) Para mostrarmos que
√
2+ 3
√
2 é irracional, primeiramente encontraremos
o polinômio mônico que tem esse número como solução. Ou seja,
x =
√
2 +
3
√
2
x−
√
2 =
3
√
2
(x−
√
2)3 = 2
x3 − 3
√
2x2 + 6x−
√
8 = 2
x3 + 6x− 2 = 3
√
2 +
√
8
(x3 + 6x− 2)2 = (3
√
2 +
√
8)2
x6 + 12x4 − 4x3 + 36x2 − 24x+ 4 = 18x4 + 24x2 + 8
x6 − 6x4 + 12x2 − 24x− 4 = 0
Logo o polinômio de coeficientes inteiros f(x) = x6 − 6x4 + 12x2 − 24x − 4
tem o número real
√
2 + 3
√
2 como sua raiz. Para concluirmos, basta utilizar
o que foi provado nos itens anteriores e observar que an = 1, logo as raízes
inteiras desse polinômio dividem a0 = −4, e como
√
2 + 3
√
2 não divide −4,
concluímos que esse número é irracional.
43. Dado um número natural p > 1, prove que os números racionais da
forma m
pn
, onde m ∈ Z e n ∈ N constituem um subconjunto denso em
R.
Resolução:
Seja (a, b) ⊂ R um intervalo qualquer. Como 0 < b− a, existem naturais p e
n tais que 0 < 1
pn
< b− a. Como os números da forma m
pn
dividem a reta em
intervalos de comprimento 1
pn
, então pelo menos um desses números pertence
ao intervalo (a, b). Seja o conjunto A = {m ∈ Z; m
pn
≥ b}. O conjunto A
é não vazio e limitado, uma vez que m ≥ b · pn e R é arquimediano. Com
isso podemos mostrar que o conjunto dos números da forma m
pn
é denso.
Seja então m0 = inf A, o número m0−1pn ∈ (a, b), do contrário teríamos
m0−1
pn
< a < b < m0
pn
, o que nos mostra que b − a < m0
pn
− m0−1
pn
= 1
pn
, um
absurdo. Portanto, o conjunto dos números racionais da forma m
pn
é denso.
19
44. Um número real r chama-se algébrico quando existe um polinômio
f(x) = a0 + a1x+ . . .+ anx
n, não identicamente nulo, com coeficientes
inteiros, tal que f(r) = 0.
i) Prove que o conjunto dos polinômios de coeficientes inteiros é enu-
merável.
ii) Dada uma enumeração {f1, f2, . . . , fn, . . .} desses polinômios não
identicamente nulos, seja, para cada n ∈ N, An o conjunto das
raízes reais de fn. Cada conjunto An é um conjunto finito (po-
dendo ser vazio). O conjunto A dos números algébricos reais
escreve-se A =
⋃
n∈N
An. Conclua que A é enumerável. Mostre
que A é denso em R.
Resolução:
i) basta tomar a função f : Zn → Pn definida por f(a0, a1, . . . , an) = a0 +
a1x+. . .+anx
n. A função f será bijetora e como Zn é enumerável, concluímos
que Pn é enumerável.
ii) Pelo exercício 20 item b do capítulo 2, o conjunto An é enumerável, e
como A =
n∈N⋃
An é reunião de conjuntos enumeráveis, então A é enumerável.
Pelo exercício 42 item ii deste capítulo, para cada intervalo (a, b), podemos
tomar um número inteiro a > 0 tal que n
√
a ∈ (a, b) e xn − a = 0, portanto
A é denso.
45. Seja X o complementar de um conjunto enumerável de números reais.
Mostre que, para cada intervalo aberto (a, b), a interseção (a, b)∩X é
não enumerável. Em particular, X é denso.
Resolução:
Como R é não enumerável, temos que X também o é, do contrário, a união
X ∪{X = R implicaria dizer que o conjunto dos números reais é enumerável.
Com isso, basta tomar valores a, b ∈ {X que a interseção (a, b)∪Xpossuirá
infinitos elementos de X.
46. Um número real chama-se transcedente quando não é algébrico. Prove
que o conjunto dos números transcedentes é não-enumerável e denso R.
Resolução:
Como R se resume à reunião do conjunto dos números algébricos com o
conjunto dos números transcendentes, e o conjunto do números algébricos é
enumerável, segundo o exercício 44 item ii, podemos simplesmente aplicar o
exercício 45 para concluir que o conjunto dos números transcendentes é não
enumerável.
20 CAPÍTULO 1. NÚMEROS REAIS
47. Prove que o conjunto dos números algébricos é um corpo. (Este ex-
ercício requer conhecimentos de Álgebra muito acima do que estamos
admitindo até aqui.)
49. Sejam B ⊂ A conjuntos não-vazios de números reais. Suponha que A
seja limitado superiormente e que, para cada x ∈ A, exista um y ∈ B
tal que x ≤ y. Prove que, nestas condições, tem-se supB = supA.
Enuncie e demonstre um resultado análogo para inf .
Resolução:
Sendo x ≤ supA para todo x de A, então devemos ter que para todo y ∈ B,
y ≤ supA. Com isso, se supA < supB, então supB não pode ser o supremo
do conjunto B, e se supA > supB, então supA não pode ser o supremo do
conjunto A. Concluímos então que supA = supB.
50. Um corte de Dedekind é um par ordenado (A, B) onde A e B são
subconjuntos não-vazios de números racionais, tais que A não possui
elemento máximo, A∪B = Q e, dados x ∈ A e y ∈ B quaisquer, tem-se
x < y.
a) Prove que, num corte de Dedekind (A, B), vale supA = inf B.
b) Seja D o conjunto dos cortes de Dedekind. Prove que existe uma
bijeção f : D → R.
Resolução:
a) Basta tomar r ∈ R tal que A = {x ∈ Q;x < r} e B = {y ∈ Q; r ≤ y}.
vemos facilmente que r = supA e que r = inf B.
b) Como visto no intem anterior que inf B = supA, basta tomar f : D → R
definida por f(A,B) = inf A. A função é fácilmente verificada como injetora
e sobrejetora.
51. Sejam X, Y conjuntos não-vazios e f : X × Y → R uma função
limitada. Para cada x0 ∈ X e cada y0 ∈ Y , ponhamos s1(x0) =
sup{f(x0, y); y ∈ Y } e s2(y0) = sup{f(x, y0);x ∈ X}. Isto define
funções s1 : X → R e s2 : Y → R. Prove que se tem sup
x∈X
s1(x) =
sup
y∈Y
s2(y). Em outras palavras
sup
x
[sup
y
f(x, y)] = sup
x
[sup
y
f(x, y)].
Resolução:
Como f é limitada, o conjunto f(X × Y ) ⊂ R é limitado. Contudo, consid-
eraremos apenas a limitação superior desse conjunto. Como s1(x0), s2(y0) ⊂
21
f(X × Y ), temos então que, se s1 ≤ s2, para cada x0k existe um yk tal
que f(x0k) ≤ f(yk) e pelo exercício 49 concluímos que s1 = s2. Analoga-
mente, se s2 ≤ s1, teremos também, pelo exercício 49, que s1 = s2. Ou seja,
como s1 = sup
y
f(x, y) e s2 = sup
x
f(x, y), podemos usar o raciocínio análogo
provamos que sup
x
[sup
y
f(x, y)] = sup
x
[sup
y
f(x, y)].
52. Enuncie e demonstre um resultado análogo ao anterio com inf em vez
de sup. Considere, em seguida, o caso "misto" e prove que
sup
y
[inf
x
f(x, y)] ≤ inf
x
[sup
y
f(x, y)]
Resolução:
Para o resultado inf
x
[inf
y
f(x, y)] = inf
x
[inf
y
f(x, y)] basta usar a demon-
stração feita no exercício anterior considerando a limitação inferior de f com
a hipótese análoga do exercício 49 em relação ao ínfimo do conjunto. Para
o resultado misto temos que considerar inicialmente o resultado obtido no
exercício 33, uma vez que para cada xok podemos obter uma infinidade de yk
de modo a termos s2k ⊂ s1k . Com isso podemos concluir que inf
x
f(x, y) ≤
sup
y
f(x, y). Para finalizarmos, aplicamos à desigualdade acima o resultado
do exercício 34 garantindo que sup
y
[inf
x
f(x, y)] ≤ inf
x
[sup
y
f(x, y)].
53. Sejam x, y números reais positivos. Prove que se tem
√
xy ≤ x+ y
2
.
Resolução:
Podemos proceder do seguinte modo
(
√
x−√y)2 ≥ 0, para todos x, y ∈ R+
x+ y − 2√xy ≥ 0
x+ y ≥ 2√xy
x+ y
2
≥ √xy
54. A desigualdade entre a média aritmética e a média geométrica, vista
no exercício anterior, vale para n números reais positivos x1, . . . , xn.
Sejam G = n√x1x2 . . . xn e A = x1+...+xnn . Tem-se G ≤ A. Isto é
22 CAPÍTULO 1. NÚMEROS REAIS
evidente quando x1 = . . . = xn. Para provar a desigualdade no caso
geral, considere a operação que consiste em substituir o menor dos
números dados, digamos xi e o maior deles, digamos xj respectivamente
por x′i =
xi·xj
G e x
′
j = G. Isto não altera a média geométrica e, quanto
à média aritmética, ela não aumenta, pois, como é fácil de se ver,
x′i+x
′
j ≤ xi+xj. Prove que repetida esta operação no máixmo n vezes,
obtemos n números iguais a G e portanto, sua média aritmética é G.
Como em cada operação a média aritmética não aumentou, conclua
que G ≤ A, ou seja, n√x1x2 . . . xn ≤ x1+...+xnn .
Resolução:
Podemos indexar os n números reais convenientemente de modo a obtermos
x
(1)
1 ≤ x
(1)
2 ≤ . . . ≤ x
(1)
n . Ao fazermos a primeira substituição teremos então
x′i =
x
(1)
1 ·x
(1)
n
G e x
′
j = G. Temos que a média geométrica não será alterada, uma
vez que
G(2) = n
√
G · x2 · . . . · xn−1 ·
x1 · xn
G
=
√
x1 · x2 · . . . · xn−1 · xn
= G
Para a média aritmética, como x1 ≤ G ≤ xn, temos que x1 − G ≤ 0 e
1− xnG ≤ 0, o que nos leva a
(x1 − G)(1−
xn
G
) ≥ 0
x1 − G −
x1 · xn
G
+ xn ≥ 0
x1 + xn ≥ x′i + x′j
x1 + x2 + . . .+ xn−1 + xn ≥ x′i + x2 + . . .+ xn−1 + x′j
x1 + x2 + . . .+ xn−1 + xn
n
≥
x′i + x2 + . . .+ xn−1 + x
′
j
n
Desse modo A ≥ A(2), com A(2) = x
′
i+x2+...+xn−1+x
′
j
n
. Ao fazermos essa sub-
stituição no máximo n vezes, obtemos uma sequência de médias aritméticas
e geométricas tais que
{
A ≥ A(2) ≥ . . . ≥ A(n)
G = G(2) = . . . = G(n) , além de obtermos n valores
G de modo que
A(n) = G + . . .+ G
n
=
n · G
n
= G
23
com isso, concluímos que G = A(n) ≤ . . . ≤ A =⇒ G ≤ A.
55. Seja K um corpo ordenado completo. Indique com 0′ e 1′ o zero e a
unidade de K. Para cada n ∈ N, sejam n′ = n · 1′ + . . . + 1′(n vezes)
e (−n)′ = −n′. Definamos uma função f : R → K pondo f
(
p
q
)
= p
′
q′
,
para todo p
q
∈ Q e para x irracional, seja f(x) = sup
{
p′
q′
∈ K; p
q
< x
}
.
Prove que f é um homomorfismo sobrejetivo e conclua que f é uma
bijeção, ou seja um isomorfismo de R sobre K.
Resolução:
Achamos que a definição n′ = n ·1′+ . . .+1′ foi digitada errada. Acreditamos
que o correto seria n′ = n·1′ = 1′+. . .+1′. Usaremos a forma que acreditamos
ser o correto. Pelo exercício 1 desse capítulo temos que a′
b′
+ c
′
d′
= a
′d′+b′c′
b′d′
e
a′
b′
· c′
d′
= a
′c′
b′d′
. então calculando
f
(a
b
+
c
d
)
= f
(
ad+ bc
bd
)
=
a′d′ + b′c′
b′d′
=
a′
b′
+
c′
d′
= f
(a
b
)
+ f
( c
d
)
e
f
(a
b
· c
d
)
= f
(ac
bd
)
=
a′c′
b′d′
=
a′
b′
· c
′
d′
= f
(a
b
)
· f
( c
d
)
para todos a
b
, c
d
∈ Q. Se x é irracional, pelo exercício 50, f(x) é um corte
de Dedekind e dados x, y irracionais distintos, temos que f(x) 6= f(y), x =
sup f(x) e y = sup f(y). Logo, f(x + y) = x + y = f(x) + f(y) e f(xy) =
xy = f(x)f(y). Temos também que pelo exercício 27 que se a é racional e
x é irracional, então ax e a + x são irracionais e vale o resultado anterior.
Assim concluímos que f é um homomorfismo sobrejetivo. Para a injetividade
usamos o exercício 4 item b, uma vez que para essa função f(1) = 1, logo a
função é bijetiva.
56. Seja f : R → R um isomorfismo de R em si mesmo. Prove que f =
identidade. Conclua que se K e L são corpos ordenados completos,
existe um isomorfismo de K sobre L.
Resolução:
O resultado sai facilmente ao definirmos uma função f : K → L como no
exercício 55, pois f será um isomorfismo (homomorfismo bijetivo). Para
concluir, aplicamos o resultado do exercício 5, onde f(x) = 0 (para todo x)
ou f(x) = x. Como f(1) = 1, então concluímos que f = identidade.
57. Verifique que f : R → (−1, 1), definida por f(x) = x√
1+x2
, é uma
bijeção de R sobre o intervalo (−1, 1).
24 CAPÍTULO 1. NÚMEROS REAIS
Resolução:
Dados x, y reais tais que f(x) = f(y), temos
x
1 + x2
=
y
1 + y2
x(1 + y2) = y(1 + x2)
x+ xy2 − y − yx2 = 0
xy2 − y + x− yx2 = 0
(y − x)(xy − 1) = 0
Portanto, x = y ou xy = 1 para todos x y ∈ R. Se xy = 1, obtemos um
absurdo ao substituirmos x ou y por 0. Logo, concluímos que x = y e que
f é injetiva. Para sobrejetividadede f , consideremos que para todo x real,
x2 ≥ 0 =⇒ x2 + 1 > x2, ou seja, x2
x2+1
< 1 =⇒
√
x2
1+x2
=
∣∣∣ x√
1+x2
∣∣∣ < 1.
Portanto, para todo x ∈ R, x√
1+x2
∈ (−1, 1). Assim, se tomarmos y ∈ (−1, 1)
teremos que |y| < 1 =⇒ 0 < 1 − y2 e basta tomar x = y
1−y2 , se x > 0, e
x = − y
1−y2 , se x < 0, mostrando que f é sobrejetiva.
58. Um conjunto G de números reais chama-se um grupo aditivo quando
0 ∈ G e x, y ∈ G =⇒ x − y ∈ G. Então, x ∈ G =⇒ −x ∈ G
e x, y ∈ G =⇒ x + y ∈ G. Seja então G ⊂ R um grupo aditivo
de números reais. Indiquemos com G+ o conjunto dos números reais
positivos pertencentes a G. Excetuando o caso trivial G = {0}, G+ é
não-vazio. Suponhamos pois G 6= {0}. Prove que:
i) Se inf G+ = 0 então G é denso em R;
ii) Se inf G+ = a > 0, então a ∈ G+ e G = {0, ±a, ±2a, . . .}.
[Sugestão: para provar (ii) note primeiro que se fosse a 6∈ G+
existiriam g, h ∈ G+ com a < h < g < a+ a
2
. donde a
2
> g − h ∈
G+, uma contradição. Em seguida, observe que todo g ∈ G se
escreve sob a forma g = a · q + r, com q ∈ Z, sendo 0 ≤ r < a.
Veja que r = g − q · a ∈ G, pois q é inteiro.]
iii) Conclua que, se α ∈ R é irracional, os números reais da forma
m+ nα, com m, n ∈ Z, constituem um subconjunto denso em R.
Resolução:
i) Sendo inf G+ = 0, então para todo x ∈ G+, 0 ≤ x. Como G 6= {0}
então existe um número real a0 ∈ G tal que a0 6= 0. Desse modo temos que
a0 ∈ G =⇒ −a0 ∈ G e G+ é não-vazio, pois a0 ∈ G+ ou −a0 ∈ G+. Seja
x ∈ R qualquer e � > 0 tal que a0 < �. Como R é arquimediano, existe n ∈ N
de modo que na0 ≤ x < na0 + a0 =⇒ 0 ≤ x − na0 < a0 < �, ou seja,
25
essa desigualdade nos mostra que |na0 − x| < � =⇒ x − � < na0 < x + �,
portanto na0 ∈ (x− �, x+ �). Assim concluímos que G é denso em R.
ii) Supondo que a 6∈ G. Seja então g ∈ G tal que a < g < a + a
2
, como
a+ a
2
não é cota inferior de G+, então subtraindo a da desigualdade anterior
obtemos 0 < g−a < a
2
. Como g também não é cota inferior de G+, podemos
tomar h ∈ G tal que a < h < g < a + a
2
. Assim, 0 < g − h, −h < −a e
g < a + a
2
nos levam a ter 0 < g − h < a
2
e como g − h ∈ G, temos que a
não pode ser cota inferior de G+, portanto, a ∈ G. Desse modo, todo g ∈ G
é da forma g = a · q + r, com q ∈ Z e r ∈ R. Ou seja, r = g − a · q e como
g, a · q ∈ G, concluímos que r ∈ G e G = {0, ±a, ±2a, . . .}.
iii) Tomando G = {x ∈ R;x = m + nα, comm, n ∈ Z}. Mostraremos que
inf G+ = 0. Do contrário, se inf G+ > 0, pelo item anterior, inf G+ ∈ G+
e G = {0, ±α, ±2α, . . .} contrariando a forma dos elementos do conjunto G,
portanto, inf G+ = 0 provando que G é denso em R.
59. Sejam f, g : R × R → R e φ, ψ : R × R → R as funções definidas
por f(x, y) = 3x − y, g(x, y) = (x − 1)2 + (y + 1)2 − 9, φ(x, y, z) =
3z, ψ(x, y, z) = x2 + y2 − z. Interpretando (x, y) como as coordenadas
cartesianas de um ponto do plano R2 e (x, y, z) como as coordenadas
cartesianas de um ponto do espaço R3, descreva geometricamente os
conjuntos f−1(0), g−1(0), φ−1(0), ψ−1(0).
Resolução:
Para descrevermos geometricamente cada conjunto devmos primeiramente
conhecer f(0), g(0), φ(0), ψ(0). Temos que f(0) = 0, g(0) = −9, φ(0) = 0
e ψ(0) = 0, desse modo f−1(0) = {(x, y) ∈ R2; 3x − y = 0}, portanto,
geometricamente, f−1(0) é representado pela reta y = 3x. Analogamente,
g−1(0) = {(x, y) ∈ R2; (x− 1)2 + (y + 1)2 = 0}, φ−1(0) = {(x, y, z) ∈ R3; z =
0} e ψ−1(0) = {(x, y, z) ∈ R3; z = x2 + y2}, assim g−1(0) é representada
geometricamente pelo ponto de coordenadas (1,−1), ou seja, é o centro de
uma circunferência de raio 0. φ−1(0) é representado geometricamente pelo
plano (x, y, 0) e ψ−1(0) é representado geometricamente por um parabolóide.
60. Seja a um número real positivo. Dado um número racional p/q (onde
p ∈ Z e q ∈ N), defina a potência de base a e expoente racional p/q
como ap/q = q
√
ap. Prove:
1º) Para quaisquer r, s ∈ Q tem-se ar · as = ar+s e (ar)s = ar·s;
2º) Para todo r ∈ Q+, a função f : (0,+∞) → (0,+∞), dada por
f(x) = xr, é uma bijeção crescente;
3º) A função g : Q→ R definida por g(r) = ar, (onde a é um número
real positivo fixado) é crescente se a > 1, e descrescente se 0 <
a < 1.
26 CAPÍTULO 1. NÚMEROS REAIS
Resolução:
1º) Dados r, s ∈ Q existem p, q, p′, q′ ∈ Z tais que r = p
q
e s = p
′
q′
. Desse
modo, temos que
ar+s = a
p
q
+ p
′
q′
= a
pq′+p′q
qq′
=
qq′
√
apq′+p′q
=
qq′
√
apq′ · ap′q
=
qq′
√
apq′ · qq
′√
ap′q
= a
p
q · a
p′
q′
= ar · as
bem como,
(ar)s = (a
p
q )
p′
q′
=
q′
√(
a
p
q
)p′
=
q′
√
a
p
q · a
p
q . . . · a
p
q , com p′ fatores
=
q′
√
a
p′p
q
= a
p′p
q
q′
= a
pp′
qq′
= ar·s
2º) Sejam x, y ∈ (0,+∞) tais que f(x) = f(y), vamos mostrar que x = y e
f é injetiva. De fato, se f(x) = f(y), então
xr = yr
x
p
q = y
p
q , p ∈ Z, q ∈ N
(x
p
q )q = (x
p
q )q
xp = yp
xp − yp = 0
(x− y)(xp−1 + xp−2y + . . .+ xyp−2 + yp−1) = 0
como x, y são positivos, temos que xp−1+xp−2y+ . . .+xyp−2+yp−1 6= 0, logo
x = y e f é injetiva. Para a bijeção basta tomar f(x) = xr = x
p
q = ( q
√
x)
p
27
e como p ≥ 0, por definição, f é sobrejetiva pelo que foi mostrado na p.82.
Portanto, f é bijetiva e pela monotonicidade da multiplicação, se 0 < x < y,
então xr < yr =⇒ f(x) < f(y), para quaisquer reais x, y. Assim concluímos
que f é bijetiva crescente.
3º) Se a > 1, ao multiplicarmos a desigualdade por ar obtemos ar+1 > ar e
como r + 1 > r, para todo r ∈ Q, temos que f é crescente. Se 0 < a < 1,
também multiplicando por ar temos que ar+1 < ar, nos mostrando que f é
descrescente.

Outros materiais